Recherche sur le blog!

Affichage des articles dont le libellé est P'. Afficher tous les articles
Affichage des articles dont le libellé est P'. Afficher tous les articles

Concours Physique École Polytechnique (P') 1995 (Énoncé)

ECOLE POLYTECHNIQUE OPTION P’
CONCOURS D'ADMISSION 1995
PREMIÈRE COMPOSITION DE PHYSIQUE (3 heures)
Ce problème traite de quelques aspects de la propagation d'impulsions lumineuses très brèves dans la silice d'une fibre optique. On supposera ce milieu diélectrique, parfaitement isolant, non magnétique \(\vec H = {\vec B}/{\mu _0}\) ,homogène et isotrope. On ne considérera dans tout le problème que des ondes qui se propagent dans ce milieu le long de l'axe Oz, ne dépendent pas de x et y, et sont polarisées linéairement le long de Ox
Intégrale
On donne \(\int_{ - \infty }^{ + \infty } {dx\;{e^{\left( { - {\alpha ^2}\,{x^2} + i\,\beta \,x} \right)}}} = \frac{{\sqrt \pi }}{\alpha }{e^{ - {\beta ^2}/{4{\alpha ^2}}}}\)
valable pour tout β réel et pour tout α complexe tel que \( - \frac{\pi }{4} < Arg\;\alpha < + \frac{\pi }{4}\) .
Équations de Maxwell
\(div\;\vec B = 0\quad r\vec ot\;\vec E = - \frac{{\partial \vec B}}{{\partial t}}\quad div\;\vec D = \rho \quad r\vec ot\;\vec H = \vec j + \frac{{\partial \vec D}}{{\partial t}}\)
Analyse vectorielle
\(r\vec ot\;r\vec ot\;\vec a = g\vec rad\;div\;\vec a - \vec \Delta \;\vec a\)
Données numériques
Vitesse de la lumière dans le vide: c = 3,00 x 108 m.s-1.
Valeurs de l'indice de réfraction n0 de la silice, ainsi que de ses dérivées \({n'_0} = {\left( {dn}/ {d \omega } \right)}_{\omega = {\omega _0}}\) et \({n''_0} = {\left( {{d^2}n} / {d{\omega ^2}} \right)}_{\omega = {\omega _0}}\) , pour diverses valeurs de la longueur d'onde dans le vide \({\lambda _0} = {2\pi c} / {\omega _0}\) .
λ0 (µm) n0 \({n'_0}\) \({n''_0}\)
1,0 1,450 6,7 x 10-18 -3,74 x 10-33
1,3 1,447 10,1 x 10-18 -14,5 x 10-33
1,5 1,444 14,6 x 10-18 -27,6 x 10-33

I

On considère un « paquet d'onde » formé d'une superposition d'ondes planes dont le champ électrique s'écrit en notation complexe :
(1) \(\vec E(z,t) = E(z,t)\;{\vec e_x} = \int_{ - \infty }^{ + \infty } {d\omega \;{\rm{E}}(\omega )\;{e^{i(kz - \omega t)}}} \;{\vec e_x}\)
On suppose que \({\rm{E}}(\omega )\) ne prend de valeurs notablement différentes de zéro que dans un intervalle de largeur δω centré sur la valeur ω0 , avec δω « ω0 . Enfin, k est une fonction de ω .
De même, le vecteur déplacement électrique \(\vec D\) correspondant s'écrit :
(2) \(\vec D(z,t) = D(z,t)\;{\vec e_x} = \int_{ - \infty }^{ + \infty } {d\omega \;{\rm{D}}(\omega )\;{e^{i(kz - \omega t)}}} \;{\vec e_x}\)
1. Établir à partir des équations de Maxwell l'équation aux dérivées partielles liant E et D. En déduire une équation pour les composantes \({\rm{E}}(\omega )\) et \({\rm{D}}(\omega )\) . On admettra que la décomposition en ondes planes telle que (1) ou (2) est unique, ce qui entraîne que l'équation trouvée reliant E et D est valable pour chacune des ondes planes composantes de E et de D.
2. La réponse du milieu au champ appliqué est linéaire :
(3) \({\rm{D}}(\omega ) = {\varepsilon _0}\;{\varepsilon _r}(\omega )\;{\rm{E}}(\omega )\)
εr(ω) est une fonction réelle positive de ω . En déduire la relation entre k et ω . Quelles sont les propriétés optiques de la silice traduites par cette relation ?
3. On appelle k0 le vecteur d'onde k(ω0) correspondant à la pulsation moyenne ω0 du paquet d'onde. On met le champ électrique E sous la forme:
(4) \(E(z,t) = u(z,t)\;{e^{i(kz - {\omega _0}t)}}\)
u(z, t) est une « enveloppe lentement variable » du champ, c’est-à-dire que les variations spatiales (ou temporelles) de u se font sur des échelles beaucoup plus grandes que la longueur d'onde moyenne λ0 (ou la période optique \({T_0} = {2\pi } / {\omega _0} \) . On peut montrer que, pour une abscisse z donnée, la fonction u(z, t) ne prend de valeurs notablement différentes de zéro que pendant un intervalle de temps dont la largeur sera notée ΔT.
Quelle est la vitesse de phase \({v_\varphi }\) du paquet d'onde, c'est-à-dire la vitesse de propagation des oscillations de « l'onde porteuse » \({e^{i(kz - {\omega _0}t)}}\) ? L'exprimer en fonction de εr(ω0) , puis de l'indice de réfraction n0 à la fréquence ω0 .
Donner l'expression de u(z, t) sous la forme d'une intégrale sur ω .
On suppose dans toute la suite du problème que k(ω) admet un développement de la forme :
(5) \(k - {k_0} = {k'_0}\,(\omega - {\omega _0}) + {{k''}_0}(\omega - {\omega _0})^{2}/ 2\)
où \({k'_0}\) et \({k''_0}\) sont des constantes.
4. On limite dans cette question le développement (5) à son premier terme.
a) En utilisant l'expression intégrale de u(z, t), montrer que u(z, t) n'est fonction que de la variable \(({k'_0}\,z - t)\) . En déduire que l'enveloppe du paquet d'onde se propage sans déformation à une certaine vitesse v0 (vitesse de groupe pour ω = ω0) que l'on reliera à \({k'_0}\) . Vérifier que ce résultat est conforme à la définition usuelle de la vitesse de groupe par la formule \({v_g} = ({d\omega } / {dk})\) .
b) Exprimer v0 en fonction de ω0 , de l'indice de réfraction n0 du milieu à la fréquence ω0, et de la dispersion d'indice \({n'_0} = ({dn} / {d\omega } )_{\omega = {\omega _0}}\) .
c) Calculer numériquement, pour λ = 1,3 µm, la différence relative \(({\upsilon _0} - {\upsilon _\varphi })/{\upsilon _\varphi }\) entre vitesse de phase et vitesse de groupe.
d) on considère un paquet d'onde de durée ΔT = 10 ps. Déterminer la distance de propagation au bout de laquelle une arche donnée de sinusoïde de l'onde porteuse s'est déplacée d'un bout à l’autre de l'enveloppe u(z, t). Dans quel sens s'effectue ce glissement de l'onde porteuse par rapport à l'enveloppe ?
5. Dans toute la suite du problème, on utilise le développement (5) complet.
a) Montrer que la vitesse de groupe vg dépend de ω . Exprimer \({k''_0}\) en fonction de ω0 , \({n'_0}\) et de la dérivée seconde \({n''_0} = ({{d^2}n} / {d\omega }^{2})_{\omega = {\omega _0}}\) .
b) Déterminer les valeurs numériques de \({k''_0}\) dans le cas de la silice dans laquelle on fait propager des paquets d'ondes ayant une longueur d'onde dans le vide successivement égale à 1,0 µm, 1,3 µm et 1,5 µm .
6. On considère maintenant, ainsi que dans les questions 7. et 8. un paquet d'onde gaussien de la forme (1), c'est-à-dire tel que :
(6) \({\rm{E}}(\omega ) = {{\rm{E}}_{\rm{0}}}\exp \left(  - (\omega - {\omega _0}){^2} / 2 (\delta \omega ){^2} \right)\)
Calculer u(0, t). Montrer qu’en z = 0, \({\left| {u(0,t)} \right|^2}\) est une impulsion dont la largeur temporelle, pour une valeur de la fonction supérieure à 1/e fois la hauteur maximale, est une quantité ΔT0 que l'on déterminera. Quelle doit-être la valeur de δω pour que ΔT0 = 10 ps ? Comparer δω et ω0.
7.a) Calculer \({\left| {u(z,t)} \right|^2}\) en un point d'abscisse z quelconque.
b) Montrer que \({\left| {u(z,t)} \right|^2}\) est un paquet d'onde gaussien dont on déterminera la vitesse de propagation du sommet ainsi que la largeur temporelle ΔT(z) définie comme à la question 6. . Donner l'origine physique de la variation de ΔT(z) au cours de la propagation et de sa dépendance par rapport au signe de \({k''_0}\) . Montrer que l’on peut retrouver l'expression asymptotique de ΔT(z) pour z grand, à un coefficient multiplicatif près, par un raisonnement simple.
Calculer numériquement ΔT(L) pour L = 100 km et pourλ0 = 1,0 µm, 1,3 µm et 1,5 µm .
c) On définit la pulsation instantanée ωi d'une onde \(E(z,t) = \left| {E(z,t)} \right|\;{e^{ - i\,\varphi (z,t)}}\) par la relation :
(7) \({\omega _i} = \frac{{\partial \varphi }}{{\partial t}}\)
Montrer que cette définition redonne la pulsation habituelle dans le cas d'une onde monochromatique. Calculer cette quantité pour le paquet d'onde gaussien d'enveloppe u(z, t). Comment évolue ωi en un point d'abscisse z donnée ? Préciser à quel instant ωi = ω0 . Expliquer qualitativement cette évolution.
8. On utilise la fibre optique pour transporter de l'information sous forme digitale. Donner une expression approchée du débit maximum d'information que l'on peut transmettre sans risque d'erreur sur une longueur L de fibre. Déterminer numériquement ce débit pour une fibre de silice de longueur L = 100 km et pour la longueur d'onde la plus favorable que l'on déterminera parmi les trois données de l'énoncé.

II

1. On considère dans le vide un réseau plan infini fonctionnant en transmission et dont le pas est d. On suppose que ce réseau diffracte uniquement dans l'ordre +1. On envoie perpendiculairement au plan du réseau une onde plane de pulsation ω0 (voir figure). Déterminer l'angle γ sous lequel l'onde transmise est diffractée.
On utilise ensuite un second réseau, identique au premier et parallèle à celui-ci, à une distance b disposé de telle sorte que le rayon transmis par l'ensemble des deux réseaux soit parallèle au rayon incident. Calculer pour ce rayon le chemin optique [IJ] et sa dérivée par rapport à ω à travers l'ensemble du système optique précédent.
2. On envoie sur ce dispositif un paquet d'onde gaussien issu d'une fibre de silice de longueur L, et qui avait la forme (6) à l'entrée de la fibre.
a) Indiquer qualitativement pourquoi on peut retrouver, après passage à travers les deux réseaux, un paquet d'onde de largeur plus faible que ΔT(L) .
b) Quel doit être le signe de \({k''_0}\) pour qu'un tel effet de recompression puisse se produire ? Pour quelle(s) longueur(s) d'onde du tableau de la page 1 obtient-on cet effet après propagation dans une fibre de silice ?
c) Déterminer la relation liant b, c, d, ω0 , \({k''_0}\) et L pour qu'on retrouve à la sortie du dispositif un paquet d'onde gaussien de largeur égale à sa largeur ΔT(0) à l'entrée de la fibre.

Concours Physique EIVP P' 1994 (Corrigé)

I.V.P. 1994 option P' Freinage d'une navette par l'atmosphère
Interaction entre deux spires.
1 Préliminaire
$p = \mu \frac{{RT}}{M}$ atmosphère en équilibre isotherme $ \Rightarrow \mu * \vec g = gra\vec d\left( p \right)$
$\frac{\partial p}{\partial z}=-\text{ }\mu *g\text{ }\xrightarrow{{}}\text{ }\frac{\partial \mu }{\partial z}=-\text{ }\mu \cdot g\frac{M}{RT}$ $\mu = {\mu _S} \cdot \exp \left( { - {\rm{ }}\frac{{Mgz}}{{RT}}} \right) = {\mu _S}\exp \left( { - {\rm{ }}\frac{z}{d}} \right)$
$d = \frac{{RT}}{{Mg}} = {\rm{ }}8000m{\rm{ }} \Rightarrow {\rm{ }}$$T = 279{\rm{ }}K$
2 Freinage vertical $\frac{dv}{d\mu }\text{ + }\frac{{{\text{C}}_{\text{1}}}\cdot d}{m}\cdot v\text{ = 0}$
2.1. $\frac{{dv}}{{dt}} = {\rm{ - }}\mu \cdot \frac{{{{\rm{C}}_{\rm{1}}}}}{{\rm{m}}} \cdot v{{\rm{ }}^2}{\rm{ = }}\frac{{{\rm{dv}}}}{{{\rm{d}}\mu }} \cdot \frac{{d\mu }}{{dh}} \cdot \frac{{dh}}{{dt}}$
avec $\frac{{dh}}{{dt}}{\rm{ }} = {\rm{ }} - v{\rm{ ; et }}\frac{{{\rm{d}}\mu }}{{{\rm{dh}}}}{\rm{ = - }}\frac{\mu }{{\rm{h}}}$
2.2 dv/v = -(C1 .d/m).dµ Ln(v/v0) = C1 .d/m.(µ0-µ) $v{\rm{ = }}{{\rm{v}}_{\rm{0}}} \cdot \exp \left( {\frac{{d.{C_1}}}{m} \cdot ({\mu _0} - \mu )} \right)$

2.3 A l'altitude h0 la masse volumique est très faible (µ0 = 4,8.10-6) et le freinage très peu efficace; dans le cadre de ce modèle très grossier la vitesse à l'arrivée au sol est très faible: v= v0* exp(-20,8) = 7,4 .10-6 m/s
bien, sur la force de freinage, qui varie comme le carré cette vitesse, n'est plus efficace; qualitativement on voit que l'efficacité du freinage passe par un maximum; il resterait à définir quantitativement cette " efficacité".
2.4 δ = -dv/dt = µ(h).C1 /m.V2 = .C1 /m.V02 µ.exp{ 2.C1 (d./m).(µ0-µ)}
il y aurait un maximum de décélération là où d δ /dµ. = 0 = [1 - µ. 2.C1 d./m.]* δ /µ
soit quand ${\mu _M}{\rm{ = }}\frac{{\rm{m}}}{{{\rm{2}} \cdot {\rm{d}} \cdot {{\rm{C}}_{\rm{1}}}}}{\rm{ }} = {\rm{ }}\frac{{{\rm{5}}{\rm{.1}}{{\rm{0}}^{\rm{3}}}}}{{{{8.10}^3}.2.10}}{\rm{ = }}\frac{{\rm{1}}}{{{\rm{32}}}}kg/{m^3}$
ce qui correspond à une altitude ${h_M}{\rm{ = d}} \cdot {\rm{Ln(}}\frac{{{\mu _{\rm{S}}}}}{{{\mu _{\rm{M}}}}}){\rm{ = 8000}}{\rm{.Ln42 = 29}}{\rm{,9km}}$
la décélération serait $\delta _M^{}{\rm{ = }}\mu _M^{} \cdot \frac{{{C_1}}}{m} \cdot V_0^2 \cdot \frac{1}{e}{\rm{ = }}\frac{{\rm{1}}}{{\rm{e}}} \cdot \frac{{V_0^2}}{{2.d}}{\rm{ = 1470 m}} \cdot {{\rm{s}}^{{\rm{ - 2}}}}$
soit, dans le cadre de ce modèle, environ 150 fois G.
2.5 Si l'on réintroduit l'attraction terrestre son effet est notable au départ car µ est faible, la vitesse va donc augmenter; on peut majorer cette augmentation en évaluant la vitesse de la navette au sol s'il n'y avait pas d'atmosphère(v '= (64.106+10.105)½ =8,06 km/s
Sous l'action de la gravitation la navette atteindrait une vitesse limite ${V_l}{\rm{ = }}\sqrt {\frac{{{\rm{m}}{\rm{.G}}}}{{\mu .{C_1}}}} $ qui correspond en h=0, où µ = 1,3kg/m3, à Vl = 62 m/s. Dans le cadre du modèle où l' on néglige la gravitation cette vitesse de 62m/s correspond à une altitude calculable par les relations du § 2.2 : µ(h) =m /.(C1 d.).Ln(8000/62) = 0,30 kg/m3, on aurait cette masse volumique donc cette vitesse(62 m/s) en h= d.Ln(1,3/0,3) soit en h = 11,6 km ,(13,2km en tenant compte de µ(h)), altitude nettement inférieure à celle (environ 30 km) où l' on prévoyait une décélération maximale de 150* G, qu'il ne faut pas faire subir à d'hypothétiques passagers. L'allure générale de la courbe v=f(h), et ses conséquences, sont donc peu modifiées.

3 Freinage sur une spirale
3.1 On projette sur la tangente à la trajectoire la relation fondamentale en "oubliant" là encore le terme de gravitation lié à la terre(m.G.cosα) : $\frac{{dv}}{{dt}} = {\rm{ - }}\mu \cdot \frac{{{{\rm{C}}_{\rm{1}}}}}{{\rm{m}}} \cdot v{{\rm{ }}^2}{\rm{ = }}\frac{{{\rm{dv}}}}{{{\rm{d}}\mu }} \cdot \frac{{d\mu }}{{dh}} \cdot \frac{{dh}}{{dt}}$
relation inchangée, mais avec dh /dt = - Vcosα $\frac{{dv}}{{d\mu }}{\rm{ + }}\frac{{{{\rm{C}}_{\rm{1}}} \cdot d}}{{m.\cos \alpha }} \cdot v{\rm{ = 0}}$
3.2 Le freinage sur l'air raréfiée de la très haute atmosphère provoque une perte lente d'énergie mécanique pour un satellite, même en orbite circulaire (α=π / 2) et donc une lente diminution de l'altitude. Ce phénomène n'est pas pris en compte ici. Donc si α=π / 2 il n'y a pratiquement pas freinage et si α=0 la rentrée est la plus "brutale".
3.3 On observera le maximum de décélération pour d δ /dµ.=[1 - µ. 2.C1 d./m.cosα.].( δ /µ)
La décélération maximale sera $\Delta _{M}^{'}=\mu _{M}^{'}.\frac{{{C}_{t}}}{m}.V_{0}^{2}.\frac{1}{e}=\frac{\cos \alpha }{e}.\frac{V_{0}^{2}}{2.d}=1470.\cos \alpha $ pour qu'elle soit inférieure à 10.G il faudra cosα < 1 / 14,7 soit π / 2 > α > π / 2 - 0,068
La longueur L de la trajectoire parcourue par la navette sera : $L{\rm{ = }}\int_{{\rm{t = 0}}}^{{{\rm{t}}_{\rm{F}}}} {{\rm{v}}{\rm{.dt}}} {\rm{ = }}\int_{{\rm{t = 0}}}^{{{\rm{t}}_{\rm{F}}}} {\frac{{{\rm{dh}}}}{{{\rm{dt}}}} \cdot \frac{{\rm{1}}}{{{\rm{cos}}\alpha }}{\rm{dt}}} {\rm{ = 14}}{\rm{,7}}{\rm{.}}{{\rm{h}}_{\rm{0}}}{\rm{ = 1470 km}}$
3.4 On néglige, lors de la descente de la navette, la perte d'énergie potentielle gravitationnelle devant la perte d'énergie cinétique 64 fois plus importante:-ΔEc =1/2.m.v²=1,6.1011 joules
Pour dissiper cette énergie on songe à la vaporisation d'une céramique; il en faudrait:
$ - \Delta {E_c}{\rm{ = }}\left( {{{\rm{I}}_{{\rm{Fus}}}} + {I_{vap}}} \right) \cdot {{\rm{m}}_{{\rm{ceram}}}}{\rm{ Soit }}{{\rm{m}}_{{\rm{ceram}}}}{\rm{ = }}\frac{{{\rm{1}}{\rm{,6}}{\rm{.1}}{{\rm{0}}^{{\rm{11}}}}}}{{{{10}^7}}}{\rm{ = 16 tonnes }}$
Cette valeur est bien sur incohérente avec la valeur, 5 tonnes, de la masse de la navette; on peut penser qu'il y a en plus évacuation de la chaleur par convection et surtout par rayonnement, l'importance de ces facteurs augmente.si la durée du vol spirale croît(α→π / 2)
Pb 2 : INTERACTION ENTRE DEUX SPIRES
1 Etude des phénomènes électromagnétiques
1.1 ${B_z}{\rm{ = }}\frac{{{\mu _{\rm{0}}}{I_1}}}{{2a}}{\left( {1 + \frac{{{a^2}}}{{{z^2}}}} \right)^{ - \frac{3}{2}}} \cong {\rm{ }}\frac{{{\mu _{\rm{0}}}{I_1}{a^2}}}{{2{z^3}}}$
1.2 L'Inductance mutuelle M entre les deux spires:
$M{\rm{ = }}\frac{{{\Phi _{{\rm{12}}}}}}{{{{\rm{I}}_{\rm{1}}}}}{\rm{ }} \cong {\rm{ }}\frac{{{\mu _{\rm{0}}}\pi {a^4}}}{{2{z^3}}}{\rm{ }}$ et${I_{2{\rm{ }}}}{\rm{ = - }}\frac{{\rm{1}}}{{\rm{R}}}\frac{{d{\Phi _{12}}}}{{dt}}{\rm{ = }}\frac{{\rm{1}}}{{\rm{R}}}\frac{{3{\mu _0}\pi {a^4}}}{{2{z^4}}}{I_1}\left( {\frac{{dz}}{{dt}}} \right)$
M≠0 alors que L1 et L2 sont nulles peut surprendre. On peut espérer qu'un candidat s'en étonnant et invoquant L2.L1 ≥ M2 aura été fortement récompensé !
1.3 . a
1°) Symétrie de révolution autour de oz ⇒ $\vec B{\rm{ }}$ indépendant de θ
2°) Le plan $M,{\vec u_r},{\vec u_\theta }$ est plan de "symétrie négative" ⇒ $\vec B{\rm{ (M) }} \in {\rm{ au plan }}M,{\vec u_r},{\vec u_\theta }$
1.3 . b $2\pi .r.dz.{B_r} + \pi .{r^2}.\frac{{\partial {B_z}}}{{\partial z}}.dz{\rm{ = 0}}$${B_r}{\rm{ = - }}\frac{{\rm{a}}}{{\rm{2}}}\frac{{\partial {B_z}}}{{\partial z}}{\rm{ = }}\frac{{\rm{a}}}{{\rm{2}}} \cdot \frac{{3{\mu _0}\pi {a^2}{I_1}}}{{2{z^4}}}$
1 3 . c $\vec F{\rm{ = }}\oint {{{\rm{I}}_{\rm{2}}}.d\vec l \wedge \vec B} {\rm{ = - }}{{\rm{\vec u}}_{\rm{z}}}.2\pi a.{B_r}$
${F_z}{\rm{ = - }}\left( {\frac{{\rm{a}}}{{\rm{2}}} \cdot \frac{{3{\mu _0}\pi {a^2}{I_1}}}{{2{z^4}}}} \right)\frac{{\left( {2\pi a} \right)}}{{\rm{R}}}\frac{{3{\mu _0}\pi {a^4}{I_1}}}{{2{z^4}}}\left( {\frac{{dz}}{{dt}}} \right){\rm{ = - }}\frac{{\rm{1}}}{{{\rm{R}}{\rm{.}}{{\rm{z}}^{\rm{8}}}}}{\left( {\frac{{3{\mu _0}\pi {a^4}}}{2}} \right)^2}\frac{{dz}}{{dt}}$
prend la forme demandée avec $k{\rm{ = }}\frac{{\rm{1}}}{{{\rm{2}}{\rm{.m}}{\rm{.R}}}} \cdot {\left( {3{\mu _0}\pi {a^4}I} \right)^2}$ Bien sur $\vec F{{\rm{ }}_{{\rm{12}}}}{\rm{ = - }}\vec F{{\rm{ }}_{{\rm{21}}}}$

2 Etude des mouvements des spires
2.1 $\begin{array}{l}\\\frac{{{d^2}{z_1}}}{{d{t^2}}} + \frac{{{d^2}{z_2}}}{{d{t^2}}}{\rm{ = 0 ; }}\frac{{{{\rm{d}}^{\rm{2}}}{z_2}}}{{d{t^2}}}{\rm{ = - }}\frac{{\rm{k}}}{{{\rm{2}}{\rm{.}}{{\rm{z}}^{\rm{8}}}}} \cdot \frac{{dz}}{{dt}}{\rm{ = - }}\frac{{{{\rm{d}}^{\rm{2}}}{z_1}}}{{d{t^2}}}{\rm{ = }}\mathop {\frac{1}{2}}\limits^{..} \frac{{{d^2}z}}{{d{t^2}}}\end{array}$
et $\frac{{dz}}{{dt}}{\rm{ = }}\frac{{\rm{k}}}{{{\rm{7}}{\rm{.}}{{\rm{z}}^{\rm{7}}}}}{\rm{ + C = }}\frac{{\rm{k}}}{{{\rm{7}}{\rm{.z}}_{\rm{0}}^{\rm{7}}}}\left( {\frac{{{\rm{z}}_{\rm{0}}^{\rm{7}}}}{{{{\rm{z}}^{\rm{7}}}}}{\rm{ - }}1} \right){\rm{ + }}{{\rm{v}}_{\rm{0}}}{\rm{ = g(z) }}$
2.2. a d²z /dt² est négatif à la date t = 0, puisque v0 > 0; mais selon la valeur de v0 et surtout du signe de (7.v0.z07-k ), deux cas sont possibles; dans le premier g(z→∝)>0
alors(7.v0.z07-k)>o,
pour g(z→∝)>0 :
Le régime permanent lorsque t→ ∝ est un mouvement uniforme à la vitesse ${v_\infty }{\rm{ = }}{{\rm{v}}_{\rm{0}}}{\rm{ }} - {\rm{ }}\frac{k}{{7.z_0^7}}$ ; les spires s'écartent indéfiniment.
2.2 . b Si maintenant g(z→∝) < 0, c'est à dire qu'alors (7.v0.z07-k) < 0, la vitesse d'éloignement, s'annule pour une valeur finie de z; c'est le point F du graphe inférieur. Les spires s'immobilisent et comme on a toujours $\mathop {\rm{z}}\limits^{ \bullet {\rm{ }} \bullet } \cdot \mathop {\rm{z}}\limits^ \bullet {\rm{ }} \le {\rm{ 0}}$, le mouvement, ne peut reprendre.
2.2 . c La courbe intermédiaire du graphe est bien la courbe séparatrice du diagramme des phases.
2.3 D'un point de vue énergétique on peut écrire que la variation de la somme de l'énergie magnétostatique et de l'énergie cinétique deux spires est égale à la somme des énergies reçues de l'extérieur c'est à dire ici au travail du générateur qui maintient le courant ${I_1}$ constant et la chaleur " reçue"de l'extérieur , algébriquement négative, contrepartie de l'effet joule. On peut aussi écrire le théorème de l'énergie cinétique: c'est à dire: variation de l'énergie cinétique des deux spires égale au travail de toutes les forces, ici les forces de Laplace sur les deux spires. Il reste une difficulté relative à l'état initial de la seconde bobine- (à t = o, il est possible de considérer ${{\rm{I}}_{\rm{2}}}$ = 0, il faudra alors un L faible mais non nul, ou ${{\rm{I}}_{\rm{2}}}$ ≠ 0, cela à quelques répercutions sur le bilan;

A noter que $\int {d(M.{I_1}.{I_2}} ){\rm{ = }}{I_1}.\Delta (M.{I_2}) = - M.{I_1}.{I_{{2_0}}} = - {\rm{ }}\frac{{7m.v_0^2}}{{12}}$, n'est pas négligeable)
si 7.v0.z07 = 2k les spires s'éloignent indéfiniment (§ 2.2.a) et dz/dt= (v0/2).(1+ z07 /z7)
Travail des forces de Laplace:
${W_L}{\rm{ = - }}\int\limits_{{\rm{z = }}{{\rm{z}}_{\rm{0}}}}^\infty {\frac{{{\rm{k}}{\rm{.m}}}}{{\rm{2}}} \cdot \frac{1}{{{z^8}}}} \cdot \frac{{{\rm{dz}}}}{{{\rm{dt}}}} \cdot {\rm{dz = - (m}}{\rm{.}}{{\rm{v}}_{\rm{0}}} \cdot \frac{{\rm{7}}}{{\rm{4}}}{\rm{)}} \cdot \frac{{{{\rm{v}}_{\rm{0}}}}}{{\rm{2}}}\int\limits_{{\rm{u = 1}}}^\infty {\left( {{{\rm{u}}^{{\rm{ - 15}}}}{\rm{ + }}{{\rm{u}}^{ - 8}}} \right)} {\rm{ du = }}\underline {{\rm{ - }}\frac{{\rm{3}}}{{{\rm{16}}}} \cdot {\rm{mv}}_{\rm{0}}^2} {\rm{ }}$
énergie joule:
${W_J}{\rm{ = }}\int\limits_{t = 0}^\infty {{\rm{R}}{\rm{.}}{{\rm{I}}_{\rm{2}}}{{(t)}^2}} {\rm{.dt = }}\int {{\rm{R}}{\rm{.}}} {\left( {\frac{{\rm{1}}}{{\rm{R}}}\frac{{3{\mu _0}\pi {a^4}}}{{2{z^4}}}{I_1}\left( {\frac{{dz}}{{dt}}} \right)} \right)^{\rm{2}}}{\rm{dt = }}\int {\frac{{{\rm{k}}{\rm{.m}}}}{{{\rm{2}}{\rm{.}}{{\rm{z}}^{\rm{8}}}}}{{\left( {\frac{{dz}}{{dt}}} \right)}^2} \cdot } {\rm{ dt = - }}{{\rm{W}}_{\rm{L}}}$
variation d'énergie cinétique:
${E_{{c_{initiale}}}}{\rm{ = 2}}{\rm{.}}\left( {\frac{{\rm{m}}}{{\rm{2}}} \cdot {{(\frac{{v_0^{}}}{2})}^2}} \right){\rm{ = }}\frac{{{\rm{m}}{\rm{.v}}_{\rm{0}}^{\rm{2}}}}{{\rm{4}}}{\rm{ ; }}{{\rm{E}}_{{c_{finale}}}}{\rm{ = 2}}{\rm{.}}\left( {\frac{{\rm{m}}}{{\rm{2}}} \cdot {{(\frac{{v_0^{}}}{4})}^2}} \right){\rm{ = }}\frac{{{\rm{m}}{\rm{.v}}_{\rm{0}}^{\rm{2}}}}{{{\rm{16}}}}{\rm{ }} \Rightarrow {\rm{ }}\underline {\Delta {{\rm{E}}_{{\rm{c }}}}{\rm{ = }}\frac{{{\rm{3}}{\rm{.m}}{\rm{.v}}_{\rm{0}}^{\rm{2}}}}{{{\rm{16}}}}} $

Concours Physique Centrale-Supélec (M, P') 1993 (Énoncé)

Centrale–Supélec, M, P’, 1993 (Physique I)
Énoncé
Ce problème comporte trois parties dont certaines questions peuvent être abordées de façon indépendante. La première partie abordera la propagation d’une onde de courant dans une ligne électrique, la deuxième précisera la structure du champ électromagnétique dans la ligne et la troisième traitera de la transmission d’une onde électromagnétique par une lame conductrice. Les données numériques sont regroupées en fin d’énoncé; on posera \(j^2 = -1\).

Onde de courant dans une ligne électrique

Une ligne électrique sans pertes est caractérisée par son coefficient d’inductance propre linéique et sa capacité linéique, respectivement notées \(L\) et \(C\). À l’abscisse \(x\) et à l’instant \(t\), on désigne par \(i(x,t)\) l’intensité du courant dans la ligne et par \(u(x,t)\) la tension entre les deux conducteurs de la ligne (cf. fig. [fig1]).
  1. Établir les deux équations différentielles liant \(i(x,t)\) et \(u(x,t)\).
  2. [I2] On cherche une solution de ces équations représentant une onde de courant de la forme \(i(x,t) = I(x) \exp \left(j \omega t\right)\) en notation complexe. Déterminer, dans ce cas, la forme la plus générale de \(i(x,t)\) et \(u(x,t)\). Exprimer en fonction des caractéristiques de la ligne la vitesse de phase \(v_\varphi\) de cette onde.
  3. La ligne, située dans l’espace \(x < 0\), s’étend jusqu’en \(x = 0\) où elle est fermée sur l’impédance \(Z_0\) (cf. fig. [fig2]). Montrer qu’il existe une valeur \(Z_c\) de \(Z_0\), appelée impédance caractéristique de la ligne telle que le rapport \(u/i\) devienne indépendant de \(x\). On exprimera \(Z_c\) en fonction de \(L\) et \(C\) et on précisera la forme de l’onde dans la ligne. Exprimer dans ce cas la puissance moyenne transportée par l’onde à l’abscisse \(x\). Que se passe-t-il physiquement en \(x = 0\)?
  4. La ligne s’étend maintenant jusqu’à \(x = + \infty\) mais on branche encore l’impédance \(Z_0 = Z_c\) en parallèle sur la ligne à l’abscisse \(x = 0\) (cf. fig. [fig3]). On s’intéresse à l’onde de courant dans la partie \(x < 0\) de la ligne.
    1. Montrer que cette onde voit en \(x = 0\) une impédance équivalente \(Z_1\) qui s’exprime très simplement en fonction de \(Z_c\).
    2. Définir et calculer le module \(r\) du coefficient de réflexion (en courant ou en tension) de l’onde en \(x = 0\).
  5. On place enfin sur la ligne précédente un court-circuit en parallèle à l’abscisse \(x = \ell\) (cf. fig. [fig4]).
    1. Quelle est la forme nécessaire de l’onde de courant entre les abscisses \(x=0\) et \(x=\ell\)?
    2. Montrer qu’il existe une valeur minimale \(\ell_0\) de \(\ell\) telle que le courant dans la partie positive de la ligne s’annule en \(x=0\). On exprimera \(\ell_0\) en fonction de la longueur d’onde \(\lambda\) de l’onde de courant dans la ligne. En déduire alors le coefficient de réflexion et la forme de l’onde dans la partie négative de la ligne.

Champ électromagnétique dans la ligne

La ligne précédente est constituée de deux rubans conducteurs parfaits, de faible épaisseur, de largeur \(a\), distants de \(b\), l’espace entre les rubans étant vide (cf. fig. [fig5]). Les rubans sont parcourus par des courants de densités surfaciques \(\vec j_s = j_s(x,t) \vec e_x\) et \(- \vec j_s\) et présentent entre leurs faces des densités surfaciques de charge \(\sigma(x,t)\) et \(- \sigma(x,t)\).
On étudie les champs \(\vec E\) et \(\vec B\) uniquement dans l’espace situé entre les rubans et on suppose que ces champs ne dépendent que l’abscisse \(x\) du point considéré et de l’instant \(t\). On néglige donc tout effet de bord.
  1. Exprimer, en fonction des constantes électromagnétiques du vide \(\varepsilon_0\) et \(\mu_0\) et des densités \(j_s\) et \(\sigma\) les champs \(\vec E(x,t)\) et \(\vec B(x,t)\) dans l’espace vide entre les rubans.
    On considère à nouveau dans toute la suite de cette partie [PartieII] une onde de courant dans la ligne, d’intensité de la forme \(i(x,t) = I \exp \left[j \left(\omega t - k x\right)\right]\) en notation complexe, où \(k\) est une constante positive et \(I\) une constante réelle.
  2. [II2] À partir des équations de Maxwell, exprimer deux relations liant \(\sigma(x,t)\) et \(i(x,t)\). En déduire la vitesse de phase \(v_\varphi\) de l’onde et montrer que la structure du champ électromagnétique est celle d’une onde plane dans le vide illimité.
  3. Déterminer l’énergie magnétique \({\mathrm{d}}\epsilon_B\) d’une tranche d’épaisseur \({\mathrm{d}}x\) de la ligne. En déduire le coefficient d’inductance propre \(L\) de la ligne.
  4. Déterminer l’énergie \({\mathrm{d}}\epsilon_E\) associée au champ électrique \(\vec E\) de la même tranche d’épaisseur \({\mathrm{d}}x\). En déduire la capacité linéique \(C\) de la ligne.
  5. Déduire des résultats précédents l’accord entre les questions [I2] et [II2] du problème quant à la vitesse de phase \(v_\varphi\).
  6. Exprimer le champ \(\vec E\) en fonction des dimensions de la ligne et de la tension \(u(x,t)\) entre les rubans. Peut-on écrire une relation de la forme \(\vec E = - {\overrightarrow{\mathrm{grad}}\,}V\) dans l’espace vide entre les rubans?
    On désire fermer la ligne sur son impédance \(Z_c\) en introduisant, entre les rubans, à l’abscisse \(x = 0\), une plaque conductrice de résistivité \(\varrho\), d’épaisseur \(e\), de largeur \(a\) et de longueur \(b\) (cf. fig. [fig6]).
  7. On considérera dans cette question que l’épaisseur \(e\) est suffisamment faible pour que l’on puisse admettre que le courant traversant la plaque soit réparti de manière uniforme.
    1. Déterminer \(Z_c\) en fonction de \(\varrho\), \(e\), \(a\) et \(b\). Montrer que la résistance \(R_c\) d’un carré de la plaque, de côté quelconque, s’exprime en fonction des seules constantes \(\varepsilon_0\) et \(\mu_0\). On appellera impédance adaptée au vide cette grandeur \(R_c\) dont on donnera la valeur numérique.
    2. On veut réaliser cette plaque avec:
      • du cuivre de résistivité \(\varrho = {1,7\cdot 10^{-8}}{\,\Omega\cdot\mathrm{m}}\);
      • du carbone de résistivité \(\varrho = {3,5\cdot 10^{-3}}{\,\Omega\cdot\mathrm{m}}\).
      Quel devrait être, dans chaque cas, l’épaisseur \(e\) de la plaque? Commenter.
  8. Déterminer le vecteur de Poynting associé à l’onde électromagnétique entre les rubans. Quelle est la puissance moyenne transportée par l’onde? Que se passe-t-il quand l’onde arrive en \(x = 0\), la ligne étant fermée par la plaque d’impédance \(Z_c\)?

Réflexion sur une plaque conductrice

On considère à présent une onde électromagnétique plane dans le vide illimité, de pulsation \(\omega\) qui a des caractéristiques identiques à celles étudiées dans la partie [PartieII]. On écrira les champs de cette onde:
\[\vec E_i = E_0 \exp \left[j \omega\left(t - \frac{x}{c}\right)\right] \vec e_y \hspace{2em} \vec B_i = \frac{E_0}{c} \exp \left[j \omega\left(t - \frac{x}{c}\right)\right] \vec e_z\]
\(c\) est la vitesse de la lumière dans le vide. À l’abscisse \(x = 0\) (cf. fig. [fig7]) on place une plaque conductrice plane infinie, orthogonale à \(\vec e_x\), de constantes électromagnétiques égales à celles du vide \(\varepsilon_0\) et \(\mu_0\), d’épaisseur \(e\) et de résistivité \(\varrho\) identiques à celles calculées dans la partie précédente: un carré de côté quelconque de la plaque a donc une résistance \(R_c\) adaptée au vide.
  1. Expliquer qualitativement pourquoi il existera pourtant une onde réfléchie sur la plaque. En vous inspirant des résultats précédents et en argumentant votre réponse, pouvez-vous indique sans calculs quel sera le module \(r\) du coefficient de réflexion de cette onde sur la plaque?
    On se propose de retrouver ce résultat directement à partir de l’étude des ondes dans le vide et la plaque. Pour ce faire, on rappelle que, moyennant l’approximation \(\varrho\varepsilon_0\omega \ll 1\) supposée ici vérifiée, le champ électrique dans la plaque conductrice est de la forme:
    \[\vec E_\varrho = \left\{A_1 \exp \left(- \frac{x}{\delta}\right) \exp \left[j \left(\omega t - \frac{x}{\delta}\right)\right] + A_2 \exp \left(\frac{x}{\delta}\right) \exp \left[j \left(\omega t + \frac{x}{\delta}\right)\right] \right\} \vec e_y\]
    \(A_1\) et \(A_2\) sont des constantes déterminées par les conditions aux limites de la plaque et \(\delta\) une distance caractéristique du conducteur et de l’onde, appelée profondeur de peau, et qui vaut \(\displaystyle \delta = \sqrt{\frac{2\varrho}{\mu_0\omega}}\).
  2. Expliquer d’où provient l’approximation indiquée et préciser le champ magnétique \(\vec B_\varrho\) associé dans la plaque. Justifier l’expression de \(\delta\).
    \(\left(\vec E_i, \vec B_i\right)\) étant l’onde incidente arrivant sur la plaque et \(\left(\vec E_\varrho, \vec B_\varrho\right)\) l’onde se propageant dans la plaque, on désigne par \(\left(\vec E_r, \vec B_r\right)\) l’onde réfléchie sur la plaque et \(\left(\vec E_t, \vec B_t\right)\) l’onde transmise dans l’espace \(x > e\).
    On écrira \(\vec E_r\) et \(\vec E_t\) sous la forme:
    \[\vec E_r = \alpha E_0 \exp \left[j \omega\left(t + \frac{x}{c}\right)\right] \vec e_y \hspace{2em} \vec E_t = \tau E_0 \exp \left[j \omega\left(t - \frac{x}{c}\right)\right] \vec e_y\]
  3. Déterminer quatre relations liant \(\alpha\), \(\tau\), \(A_1\) et \(A_2\).
  4. Montrer que l’approximation précédente implique également qu’on ait \(e \ll \delta\). En déduire, après simplifications des relations, la valeur de \(\alpha\).
  5. Que faudrait-il placer, et à quel endroit, pour annuler l’onde réfléchie? On pourra d’abord répondre qualitativement en s’appuyant sur des résultats précédents et démontrer ensuite le résultat recherché.
Formulaire et données numériques:
Formule d’analyse vectorielle \({\overrightarrow{\mathrm{rot}}\,}{\overrightarrow{\mathrm{rot}}\,}\vec u = {\overrightarrow{\mathrm{grad}}\,}{\mathrm{div}\,}\vec u - \Delta \vec u\)
Célérité de la lumière dans le vide \(c = {3,00\cdot 10^{8}}{\,\mathrm{m}\cdot\mathrm{s}^{-1}}\)
Perméabilité magnétique du vide \(\mu_0 = {4\cdot\pi\cdot 10^{-7}}{\,\mathrm{H}\cdot\mathrm{m}^{-1}}\)

Concours Physique ENSI (P') 1992 Physique 2 (Corrigé)

Préliminaire.

Soit une machine stationnaire traversée par un écoulement stationnaire d’un fluide et qui reçoit le travail $W'$ et la chaleur $Q$. Appliquons le premier principe au système formé par cette machine et par le fluide qui dans l’état initial est en partie dans la machine et en partie à l’entrée et dans l’état final est en partie dans la machine et en partie à la sortie. Notons par l’indice e le fluide à l’entrée et par l’indice s le fluide à la sortie. Soit ${S_e}$ et ${S_s}$ les sections du tuyau à l’entrée et à la sortie et ${L_e}$ et ${L_s}$ les longueurs qu’y occupe le fluide dans l’état initial et dans l’état final. Outre $W'$, le système reçoit le travail des forces de pression à l’entrée ${p_e}{S_e}{L_e}$, le travail des forces de pression à la sortie $ - {p_s}{S_s}{L_s}$ et le travail du poids qui est $mg({z_e} - {z_s})$ parce que le poids dérive d’une énergie potentielle $mgz$ et parce que la machine et l’écoulement sont stationnaires, ce qui fait que l’énergie potentielle qu’ils stockent est constante.

$\Delta (U + {E_c}) = W' + Q + {p_e}{S_e}{L_e} - {p_s}{S_s}{L_s} + mg({z_e} - {z_s})$
Comme l’écoulement et la machine sont dans un état stationnaire, l’énergie stockée dans la machine ne varie pas, donc $\Delta (U + {E_c}) = [U + {E_c}]_e^s$. D’autre part ${H_e} = {U_e} + {p_e}{S_e}{L_e}$ et ${H_s} = {U_s} + {p_s}{S_s}{L_s}$. En divisant par la masse $m$ de fluide transvasée et en représentant par des minuscules les grandeurs massiques : $[h + \frac{1}{2}{v^2} + gz]_e^s = w' + q$ , soit en négligeant les énergies potentielle et cinétique macroscopiques :${h_s} - {h_e} = w' + q$
1. Compresseur.
1. 1. et 1. 2. La transformation est une adiabatique réversible d’un gaz parfait, qui obéit à la loi de Laplace :
$\begin{array}{l}\frac{{{T_3}}}{{{p_1}^{1 - \frac{1}{\gamma }}}} = \frac{{{T_2}}}{{{p_2}^{1 - \frac{1}{\gamma }}}} \Rightarrow {T_3} = {T_2}{\left( {\frac{{{p_1}}}{{{p_2}}}} \right)^{1 - \frac{1}{\gamma }}} = 273{(12,65/5)^{1 - \frac{1}{{1,2}}}} = 318,7K\\{c_{pg}} = \frac{{\gamma R}}{{(\gamma - 1)M}} = \frac{{1,2 \times 8,31}}{{0,2 \times 0,0865}} = 576,4{\mathop{\rm J}\nolimits} .{K^{ - 1}}.k{g^{ - 1}}\\w' = {h_s} - {h_e} = {c_{pg}}({T_3} - {T_2}) = 576,4 \times (318,7 - 273) = 26300J/kg\end{array}$
1.3. Comme la transformation est adiabatique et réversible, $\Delta s = 0$.
1.4.a. Comme pour un gaz parfait $s - {s_0} = {c_{pg}}\ln \frac{T}{{{T_0}}} - \frac{R}{M}\ln \frac{p}{{{p_0}}}{\rm{ }}$, l’équation de l’isobare de pression $p$ est : $T = {T_0}{\left( {\frac{p}{{{p_0}}}} \right)^{1 - \frac{1}{\gamma }}}\exp \frac{{s - {s_0}}}{{{c_{pg}}}}$
L’isobare de pression ${p_2}$ se déduit de l’isobare de pression ${p_1}$ par translation parallèle à l’axe des $s$ de $ - \frac{R}{M}\ln \frac{{{p_2}}}{{{p_1}}}$ puisque $s(T,{p_2}) = s(T,{p_1}) - \frac{R}{M}\ln \frac{{{p_2}}}{{{p_1}}}$.
1.4.b. Le travail du compresseur est la variation d’enthalpie dans la transformation : $({T_2},{p_2}) \to ({T_3},{p_1})$. D’après la deuxième loi de Joule, $h$ ne dépend pas de $p$ :
$w' = h({T_3},{p_1}) - h({T_2},{p_2}) = h({T_3},p) - h({T_2},p)$
Or, pour une isobare, $dh = Tds \Rightarrow h({T_3},p) - h({T_2},p) = \int\limits_{{T_2}}^{{T_3}} {Tds} $ le long de l’isobare $p$ ; $w'$ est donc l’aire hachurée entre l’isobare $p$ et l’axe des $s$ dans la figure qui suit :

$q < \oint {T\,ds} $ et $w'$ est supérieur à l’aire du cycle.

2. Condenseur.
2.1. ${q_1} = {h_g}({T_1}) - {h_g}({T_3}) + {h_L}({T_1}) - {h_g}({T_1}) = {c_{pg}}({T_1} - {T_3}) - {L_v}({T_1}) = 576,4 \times (305 - 318,7) - 175000 = - 182900{\mathop{\rm J}\nolimits} .k{g^{ - 1}}$
2.2.$\Delta s = {c_{pg}}\ln \frac{{{T_1}}}{{{T_3}}} - \frac{{{L_v}({T_1})}}{{{T_1}}} = 576,4\ln \frac{{305}}{{318,7}} - \frac{{175000}}{{305}} = - 599,1{\mathop{\rm J}\nolimits} .{K^{ - 1}}.k{g^{ - 1}}$
3. Détendeur.
3.1. Cela résulte du premier principe et de l’absence de travail et de chaleur.
3.2. On peut calculer les variations des fonctions d’état sur le chemin : $({\text{liquide à }}{T_1},{p_1}) \to ({\text{liquide à }}{T_2},{p_2}) \to ({\text{fraction }}x{\text{ gazeuse et fraction }}1 - x{\text{ liquide à }}{T_2},{p_2})$
Les propriétés d’un liquide dépendent peu de la pression : on peut calculer sa variation d’enthalpie sans se préoccuper de la pression.
$\begin{array}{l}\Delta h = {c_L}({T_2} - {T_1}) + x{L_v}({T_2}) = 0\\x = \frac{{{c_L}({T_1} - {T_2})}}{{{L_v}({T_2})}} = \frac{{1,38 \times (305 - 273)}}{{205}} = 0,2154\end{array}$
3.3. $\Delta s = {c_L}\ln \frac{{{T_2}}}{{{T_1}}} + \frac{{x{L_v}({T_2})}}{{{T_2}}} = 1380\ln \frac{{273}}{{305}} + \frac{{0,2154 \times 205000}}{{273}} = 8,8{\mathop{\rm J}\nolimits} .{K^{ - 1}}.k{g^{ - 1}}$
4. Evaporateur.
4.1. ${q_2} = (1 - x){L_v}({T_2}) = (1 - 0,2154) \times 205000 = 160800{\mathop{\rm J}\nolimits} .k{g^{ - 1}}$
4.2. $\Delta s = \frac{{(1 - x){L_v}({T_2})}}{{{T_2}}} = 589,1{\mathop{\rm J}\nolimits} .{K^{ - 1}}.k{g^{ - 1}}$
5. L’efficacité est le rapport du gain, c’est-à-dire la chaleur communiquée à l’habitation à chauffer $ - {q_1}$, au coût, qui est l’énergie à donner au compresseur $\frac{{w'}}{r}$. C’est donc $e = r\frac{{ - {q_1}}}{{w'}} = 0,8\frac{{182900}}{{26300}} = 5,56$

On peut améliorer le rendement si on peut rapprocher les températures ${T_1}$ et ${T_2}$. Tel quel, ce chauffage est 5,56 fois moins coûteux en énergie que le chauffage électrique.
6.1. Pour un cycle, le premier principe s’écrit $w' + {q_1} + {q_2} = 0$. Or la même somme calculée avec les valeurs trouvées au fil de ce problème donne $w' + {q_1} + {q_2} = 26300 - 182900 + 160800 = 4200{\mathop{\rm J}\nolimits} .k{g^{ - 1}}$ ; la vérification est acceptable à la rigueur (erreur 2 %).
Les données du problème sont-elles cohérentes ? Elles devraient vérifier $\frac{{d{L_v}}}{{dT}} = \frac{{d{h_g}}}{{dT}} - \frac{{d{h_L}}}{{dT}} = {c_{pg}} - {c_L}$. Or $\frac{{{L_v}({T_1}) - {L_v}({T_2})}}{{{T_1} - {T_2}}} = \frac{{175000 - 205000}}{{305 - 273}} = - 937,5{\mathop{\rm J}\nolimits} .{K^{ - 1}}.k{g^{ - 1}}$ , tandis que ${c_{pg}} - {c_L} = 576,4 - 1380 = - 803,6{\mathop{\rm J}\nolimits} .{K^{ - 1}}.k{g^{ - 1}}$
6.2. Comme l’entropie est une fonction d’état, la somme des variations d’entropie lors du cycle devrait être nulle. Or, elle vaut $\sum {\Delta s} = - 599,1 + 8,8 + 589,1 = - 1,2{\mathop{\rm J}\nolimits} .{K^{ - 1}}.k{g^{ - 1}}$ ; la vérification est acceptable (erreur 0,2 %).
7.1.
$\begin{array}{l}kdT = - ak(T - {T_e})dt\\adt = - \frac{{dT}}{{T - {T_e}}}\\at = - \int\limits_{{T_4}}^{{T_5}} {\frac{{dT}}{{T - {T_e}}}} \Rightarrow a = \frac{1}{t}\ln \frac{{{T_4} - {T_e}}}{{{T_5} - {T_e}}} = \frac{1}{{14400}}\ln \frac{{20}}{{10}} = {4,81.10^{ - 5}}{{\mathop{\rm s}\nolimits} ^{ - 1}}\end{array}$
7.2. ${P_e} = \frac{{ak({T_4} - {T_e})}}{e} = \frac{{{{4,81.10}^{ - 5}} \times {{2.10}^7} \times 20}}{{5,56}} = \frac{{19250}}{{5,56}} = 3460{\mathop{\rm W}\nolimits} $
7.3. La puissance rejetée dans la source froide est ${P_2} = \frac{{160840}}{{182900}}19250 = 16930W$.
Le débit d’eau froide est donc $\frac{{{P_2}}}{{{c_f}\Delta T}} = \frac{{16930}}{{4180 \times 4}} = 1,013{\mathop{\rm kg}\nolimits} .{s^{ - 1}}$.

Concours Physique Centrale-Supélec (M, P') 1991 Physique II (Corrigé)

Corrigé centrale 91 M-P'
Première partie.
I- Collision neutron-noyau
1/ Conservation de la qdm : $m{\vec V_1} = m{\vec V_2} + M{\vec w_2} \Rightarrow {\vec V_1} = {\vec V_2} + A{\vec w_2}$
Conservation de l'énergie: $\text{ }\!\!{\scriptscriptstyle 1\!/\!{ }_2}\!\!\text{ }m\vec{V}_{1}^{2}=\text{ }\!\!{\scriptscriptstyle 1\!/\!{ }_2}\!\!\text{ }m\vec{V}_{2}^{2}+\text{ }\!\!{\scriptscriptstyle 1\!/\!{ }_2}\!\!\text{ }M\vec{w}_{2}^{2}\Rightarrow \vec{V}_{1}^{2}=\vec{V}_{2}^{2}+A\vec{w}_{2}^{2}$
2/ De ${\vec V_1} = {\vec V_2} + A{\vec w_2}$, on tire : $\vec V_2^2 = {({\vec V_1} - A{\vec w_2})^2} = \vec V_1^2 + A\vec w_2^2 - 2A{V_1}{w_2}\cos \theta $
Soit $\cos \theta = \frac{{\vec V_1^2 - \vec V_2^2 + {A^2}\vec w_2^2}}{{2A{V_1}{w_2}}} = \frac{{A\vec w_2^2 + {A^2}\vec w_2^2}}{{2A{V_1}{w_2}}} = \frac{{{w_2}}}{{{V_1}}}\frac{{1 + A}}{2}$> 0 donc 0 < θ < π/2
En fonction des énergies : $\text{ }\!\!{\scriptscriptstyle 1\!/\!{ }_2}\!\!\text{ }m\vec{V}_{1}^{2}=\text{ }\!\!{\scriptscriptstyle 1\!/\!{ }_2}\!\!\text{ }m\vec{V}_{2}^{2}+\text{ }\!\!{\scriptscriptstyle 1\!/\!{ }_2}\!\!\text{ }M\vec{w}_{2}^{2}\Rightarrow {{E}_{1}}-{{E}_{2}}=\text{ }\!\!{\scriptscriptstyle 1\!/\!{ }_2}\!\!\text{ A}\,\text{m}\,\vec{w}_{2}^{2}$ et ${{E}_{1}}=\text{ }\!\!{\scriptscriptstyle 1\!/\!{ }_2}\!\!\text{ }\,\text{m}\,\vec{V}_{1}^{2}$
Alors $\cos \theta = \frac{{{w_2}}}{{{V_1}}}\frac{{1 + A}}{2} = \sqrt {\frac{{{E_1} - {E_2}}}{{A{E_1}}}} \frac{{1 + A}}{2}$donc $\frac{{{E_2}}}{{{E_1}}} = 1 - \frac{{4A{{\cos }^2}\theta }}{{{{(1 + A)}^2}}}$

II- Modèle des sphères dures.
1/ La force de contact passe par le centre d'inertie, donc la vitesse ${\vec w_2}$ sera dirigé suivant la réaction normale. On en déduit : $\sin \theta = \frac{b}{{{R_1} + {R_2}}}$
2/ Le paramètre d'impact peut varier entre 0 et la valeur R1 + R2. Ce qui correspond pour le centre du neutron à
à une cible de surface variant de 0 à (R1 + R2)2.
La probablité de recevoir un impact sur une couronne de rayon : b → b + db est :$\frac{{dP}}{1} = \frac{{2\pi bdb}}{{\pi {{({R_1} + {R_2})}^2}}}$
3/ Par définition: $ < - Ln\,[1 - K{\cos ^2}\theta ]{ > _b} = < - Ln\,[1 - \frac{{K{b^2}}}{{{{({R_1} + {R_2})}^2}}}]{ > _b} = - \int\limits_0^{{R_1} + {R_2}} {Ln[1 - \frac{{K{b^2}}}{{{{({R_1} + {R_2})}^2}}}]\;db} $
En posant $x = \frac{{K{b^2}}}{{{{({R_1} + {R_2})}^2}}}$⇒ $\frac{1}{K}\left[ {(1 - x)Ln(1 - x) - (1 - x)} \right]_0^K = \frac{1}{K}\left[ {(1 - K)Ln(1 - K) - (1 - K) + 1} \right]$
Ce qui donne : $1 + \frac{{1 - K}}{K}Ln(1 - K)$ cqfd . Il faut que 0 < K < 1 pour que la fonction aît un sens.
4/ On a obtenu $\frac{{{E_2}}}{{{E_1}}} = 1 - \frac{{4A{{\cos }^2}\theta }}{{{{(1 + A)}^2}}} = 1 - K{\cos ^2}\theta $ avec $K = \frac{{4A}}{{{{(1 + A)}^2}}}$< 1 si A > 1
on peut utiliser le résultat précédent : $K = \frac{{4A}}{{{{(1 + A)}^2}}} \Rightarrow 1 - K = {\left( {\frac{{A - 1}}{{A + 1}}} \right)^2}$
Donc coefficient de ralentissement : $\gamma = < - Ln\,[\frac{{{E_2}}}{{{E_1}}}]{ > _b} = 1 + {\left( {\frac{{1 - A}}{{\sqrt {2A} }}} \right)^2}Ln(\frac{{A - 1}}{{A + 1}}) = $
5/ a)La dérivée de γ vaut zéro pour : $0 = \left( {\frac{{1 - A}}{{\sqrt A }}} \right)\left\{ { - \left( {\frac{{{A^{1/2}} + {A^{ - 1/2}}}}{{2\sqrt 2 \;A}}} \right)Ln(\frac{{A - 1}}{{A + 1}}) - \left( {\frac{1}{{\sqrt A }}} \right)\left( {\frac{1}{{(A + 1)}}} \right)} \right\}$
Le terme entre crochet ne s'annulant pas, la racine est A = 1. On vérifiera que c'est bien un maximum pour le ralentissement.
b) A-N : 1H (A = 1) γ = 1 ; 2H (A = 2) γ = 0,725 ; 12C (A = 12) γ = 0,158 ; 238U (A = 238) γ = 0,008 ;
III- Application aux ralentissements des neutrons.
1/ Il y a ½ kT par degré de liberté, donc E300K = 3/2kT = 3,9.10−2 eV.
C'est très faible devant l'énergie initiale des neutrons. On peut considèrer les noyaux immobiles, sauf pour les dernières collisions.
2 a/ Avec $\gamma = < - Ln\,[\frac{{{E_2}}}{{{E_1}}}]{ > _b}$ et en écrivant : $\frac{{{E_n}}}{{{E_0}}} = \frac{{{E_n}}}{{{E_{n - 1}}}}\frac{{{E_{n - 1}}}}{{{E_{n - 2}}}}\; \cdots \frac{{{E_1}}}{{{E_0}}} \Rightarrow Ln\left( {\frac{{{E_n}}}{{{E_0}}}} \right) = \sum\limits_1^n {Ln\left( {\frac{{{E_p}}}{{{E_{p - 1}}}}} \right)} $
on a en raisonnant sur les valeurs moyennes : $Ln\left( {\frac{{{E_n}}}{{{E_0}}}} \right) = - n\gamma \Rightarrow {E_n} = {E_0}{e^{ - \gamma }}$
2b/ $n = - \frac{1}{\gamma }Ln\left( {\frac{{{E_{300K}}}}{{{E_0}}}} \right)$d'où 1H : n = 17 ; 2H : n = 24 ; 12C : n = 108 ; 238U : n = 214;
3a/ A une date t : $v(t) = \sqrt {\frac{{2E(t)}}{m}} $, la durée moyenne intercollision est: $\Delta t = \frac{\lambda }{{v(t)}}$et le nombre de collisions par unité de temps est : $\frac{{dn}}{{dt}} = \frac{1}{{\Delta t}} \Rightarrow \frac{{dn}}{{dt}} = \frac{1}{\lambda }\sqrt {\frac{{2E}}{m}} $.
3b/ L'équation $Ln\left( {\frac{{{E_n}}}{{{E_0}}}} \right) = - n\gamma $donne, en passant à la limite : $\gamma \,dn = - Ln\,[\frac{{E + dE}}{E}] = - \frac{{dE}}{E}$
soit : $\gamma \frac{{dt}}{\lambda }\sqrt {\frac{{2E}}{m}} = - \frac{{dE}}{E}$ ; en posant $\,y = \frac{E}{{\;{E_0}}}$ on a $\gamma \frac{{dt}}{\lambda }\sqrt {\frac{{2{E_0}}}{m}} = - \frac{{dy}}{{\;{y^{3/2}}}}$
3c/ L'intégration conduit à : $2\left[ {{y^{ - 1/2}} - 1} \right] = \frac{\gamma }{\lambda }t\,\sqrt {\frac{{2{E_0}}}{m}} $soit : $\,\sqrt {\frac{{{E_0}}}{E}} = 1 + \frac{\gamma }{{2\lambda }}t\,\sqrt {\frac{{2{E_0}}}{m}} $
4a/ On calcule d'abord $\,\sqrt {\frac{{{E_0}}}{E}} \approx 5000$ puis avec γ = 0,158 on trouve t = 120 µs .
On a toujours : $\,\sqrt {\frac{{{E_0}}}{E}} > > 1$ donc $\,t = \frac{{2\lambda }}{\gamma }\sqrt {\frac{m}{{2E}}} $ indépendant de E0.
4b/ La distance parcourue pendant dt est : $dx = v.dt = dt\sqrt {\frac{{2E}}{m}} $ et on a aussi $\gamma \frac{{dt}}{\lambda }\sqrt {\frac{{2E}}{m}} = - \frac{{dE}}{E}$
donc $dx = - \frac{\lambda }{\gamma }\frac{{dE}}{E} \Rightarrow x = \frac{\lambda }{\gamma }Ln\,{\frac{{{E_0}}}{E}_{300K}}$ on trouve ainsi x = 2,8 m.
On peut remarquer que cette distance corespond à nλ puisque $n = - \frac{1}{\gamma }Ln\left( {\frac{{{E_{300K}}}}{{{E_0}}}} \right)$.

Deuxième partie.
1a/ Avec $\xi \,\vec u = {A_1}M \to $ ⇒ le théorème d'Ampère donne$\vec B = \frac{{{\mu _0}I}}{{2\pi {\xi ^2}}}\vec k \wedge \xi \vec u$
1b/${A_1}M \to = $$(r - a\cos \theta ){\vec u_r} + a\sin \theta {\vec u_{^\theta }}$⇒$\vec B = \frac{{{B_0}}}{{{\xi ^2}}}\left\{ \begin{array}{l} - a\sin \theta \;{{\vec u}_r}\\(r - a\cos \theta \;){{\vec u}_\theta }\end{array} \right.$et${\xi ^2} = {a^2} + {r^2} - 2\,a\,r\cos \theta $
1c/ $\vec B' = {B_0}\left\{ \begin{array}{l} - \frac{{a\sin \theta }}{{{\xi ^2}}}\; = - \left[ {\sin \theta {\rm{ + 2}}u\sin \theta \;\cos \theta - {{\rm{u}}^{\rm{2}}}\sin \theta [1 - 4{{\cos }^2}\theta {\rm{]}}} \right]{\rm{ }}\\\frac{{(r - a\cos \theta \;)}}{{{\xi ^2}}} = \left[ {u - {\rm{cos}}\theta - 2{\rm{u}}\,{\rm{co}}{{\rm{s}}^{\rm{2}}}\theta + {{\rm{u}}^{\rm{2}}}\cos \theta [3 - 4{{\cos }^2}\theta ]} \right]\end{array} \right.$
2a/ Il faut faire une rotation de π et changer le signe du courant. Soit: $\vec{B}''(u,\theta )=-\vec{B}'(u,\theta +\pi )$
2b/ ${B_{1r}} = B{'_r}(u,\theta ) - B{'_r}(u,\theta + \pi ) = - 2{B_0}\left[ {\sin \theta - {{\rm{u}}^{\rm{2}}}\sin \theta [1 - 4{{\cos }^2}\theta {\rm{]}}} \right]$
${B_{1\theta }} = B{'_\theta }(u,\theta ) - B{'_\theta }(u,\theta + \pi ) = - 2{B_0}\left[ {{\rm{cos}}\theta - {{\rm{u}}^{\rm{2}}}\cos \theta [3 - 4{{\cos }^2}\theta ]} \right]$
en linéarisant : ${B_{1r}} = - 2{B_0}\left[ {\sin \theta + {{\rm{u}}^{\rm{2}}}\sin 3\theta } \right]$ et${B_{1\theta }} = - 2{B_0}\left[ {{\rm{cos}}\theta + {{\rm{u}}^{\rm{2}}}\cos 3\theta } \right]$
3a/ Il faut faire une rotation d'angle − 2π/3 et d'angle +2π/3 .
3b/ Donc ${B_r} = {B_{1r}}(u,\theta ) + {B_{1r}}(u,\theta - 2\pi /3) + {B_{1r}}(u,\theta + 2\pi /3)$
${B_\theta } = {B_{1\theta }}(u,\theta ) + {B_{1\theta }}(u,\theta - 2\pi /3) + {B_{1\theta }}(u,\theta + 2\pi /3)$
Or $\left\{ \begin{array}{l}\cos (\theta - 2\pi /3) + \cos (\theta + 2\pi /3) = - \cos \theta \\\sin (\theta - 2\pi /3) + \sin (\theta + 2\pi /3) = - \sin \theta \end{array} \right.$on a finalement:
${B_r} = - 2{B_0}\left[ {3{{\rm{u}}^{\rm{2}}}\sin 3\theta } \right]$
${B_\theta } = - 2{B_0}\left[ {3{{\rm{u}}^{\rm{2}}}\cos 3\theta } \right]$ donc $C = 6$
4a/ Ligne de champ: $d\vec \ell //\vec B \Rightarrow \frac{{dr}}{{rd\theta }} = \frac{{{B_r}}}{{{B_\theta }}}$ ⇒$\frac{{dr}}{r} = \frac{{\sin 3\theta }}{{\cos 3\theta }}d\theta \Rightarrow \,{r^3} = r_0^3/\cos 3\theta $
4b/ ci-contre : allure des lignes de champ.
4c/ Module $B(r) = 6{B_0}\;{r^2}/{a^2}$,
lignes isomodules B(r) = Cte sur un cercle de centre O
II- Action du champ sur un neutron
1a/ Pour un dipôle donc deux cas possibles : ${{E}_{//}}=-\,B$ et ${{E}_{\bot }}=\,B$
Soit en remplaçant B par $C{B_0}\;{r^2}/{a^2}$⇒ ${{E}_{//}}=-\text{ }\!\!{\scriptscriptstyle 1\!/\!{ }_2}\!\!\text{ }m{{\Omega }^{2}}{{r}^{2}}$ et ${{E}_{}}=\text{ }\!\!{\scriptscriptstyle 1\!/\!{ }_2}\!\!\text{ }m{{\Omega }^{2}}{{r}^{2}}$
1b/ La force est donnée par : $\vec F = \, - gr\vec ad\,{E_p}$ donc ${\vec F_{//}} = m{\Omega ^2}\,\vec r$ et ${\vec F_{\rlap{--} \rlap{--} \not /\rlap{--} /}} = - m{\Omega ^2}\,\vec r$
Pour confiner il faut une force de rappel, seuls les neutrons antiparallèles peuvent être confinés.
2a/ La RFD donne : ${\vec F_{\rlap{--} \rlap{--} \not /\rlap{--} /}} = - m{\Omega ^2}\,\vec r = m\frac{{{d^2}\vec r}}{{d{t^2}}} + m\frac{{{d^2}z}}{{d{t^2}}}\vec k$ ⇒$ - m{\Omega ^2}\,\vec r = m\frac{{{d^2}\vec r}}{{d{t^2}}}{\rm{ et }}\frac{{{d^2}z}}{{d{t^2}}} = 0$
2b/ L'intégration donne :$\,\vec r(t) = {\vec A_1}\cos \,\Omega t + {\vec A_2}\sin \Omega t$ où ${\vec A_1}{\rm{ et }}{\vec A_2}$ sont des constantes.
soit avec les conditions initiales: $z = {v_0}t$ et $\,\vec r(t) = {x_0}\vec i\cos \,\Omega t + \frac{{{u_0}}}{\Omega }\vec j\sin \Omega t$.
2c/ La trajectoire est une hélice d'axe Oz et de section elliptique.
3a/ Le neutron est confiné si le grand axe de l'ellipse est inférieur au rayon a; x0 étant plus petit que a il faut que:$a > \frac{{{u_0}}}{\Omega }$ soit encore :${u_C} = a\,\Omega $.
3b/ A-N: uC = 5,9 m.s−1ce qui donne EC = 18.10−8 eV et aussi TC = 1,4.10−3 K
Ce résultat justifie l'appellation neutron ultra-froids.
3c/ La fonction de répartition de Boltzmann permet de calculer la fraction de neutrons qui ont une énergie inférieure à la valeur calculée précédemment:
$F = \int\limits_0^{{E_C}} {\frac{1}{{\sqrt {2\pi } }}\frac{1}{{{{(kT)}^{3/2}}}}\sqrt E \exp ( - E/kT)\,dE} $
si T = 300 K << TC on peut simplifier ⇒$F \approx \int\limits_0^{{E_C}} {\frac{1}{{\sqrt {2\pi } }}\frac{1}{{{{(kT)}^{3/2}}}}\sqrt E \,dE} = \frac{1}{{\sqrt {2\pi } }}\frac{1}{{{{(kT)}^{3/2}}}}\frac{2}{3}\left[ {{E^{3/2}}} \right]_0^{{E_C}}$
Soit finalement : $F = \sqrt {\frac{3}{{4\pi }}} {\left[ {\frac{{{T_C}}}{T}} \right]^{3/2}} \approx {5.10^{ - 9}}$ donc extrémement faible.
4/ Les neutrons ont un mouvement de dérive suivant l'axe Oz. or les fils créant le champ magnétique ne peuvent être rééllement infinis. Le confinement n'a lieu que dans la partie centrale du dispositif et se termine lorsque les neutrons sortent du dispositif.
III- Amélioration du confinement

1a/ Pour les neutrons confinés : ${\vec F_{//}} = - m{\Omega ^2}\,\vec r$ avec maintenant $\vec r = $$O'M \to $$ = (\rho - R){\vec u_\rho } + z\vec k$
1b/ En cylindriques : $\vec a = (\ddot \rho - \rho {\dot \theta ^2}){\vec u_\rho } + (2\dot \rho \dot \theta + \rho \ddot \theta ){\vec u_\theta } + \ddot z\vec k$
1c/ Equations du mouvement : $\left\{ \begin{array}{l}\ddot \rho - \rho {{\dot \theta }^2} = - {\Omega ^2}(\rho - R)\\2\dot \rho \dot \theta + \rho \ddot \theta = 0\\\ddot z = - {\Omega ^2}z\end{array} \right.$
2a/ Compte tenu des conditions initiales: $\ddot z = - {\Omega ^2}z \Rightarrow z = {z_0}\cos (\Omega t) + \frac{{{V_0}}}{\Omega }\sin \Omega t$.
2b/ $2\dot \rho \dot \theta + \rho \ddot \theta = \frac{1}{\rho }\frac{{d({\rho ^2}\dot \theta )}}{{dt}} = 0 \Rightarrow {\rho ^2}\dot \theta = Cte = \rho _0^2{\omega _0}$ "mouvement projeté sur x0y à force centrale".
2c/ Il reste l'équation en ρ(t): $\ddot \rho - \rho {\dot \theta ^2} = \ddot \rho - \left( {\frac{{\rho _0^4\omega _0^2}}{{{\rho ^3}}}} \right) = - {\Omega ^2}(\rho - R)$
3a/ si ω0 = 0 alors θ = θ0 est constant : $\ddot \rho = - {\Omega ^2}(\rho - R) \Rightarrow (\rho - R) = ({\rho _0} - R)\cos \Omega t$,
c'est l'équation paramètrique (z(t),ρ(t)) d'une ellipse de centre O'.
3b/ si $\dot \theta = Cte = {\omega _0}$ alors ${\rho ^2} = \rho _0^2$, la trajectoire est sinusoïde dessinée sur un cylindre d'axe Oz.
La trajectoire sera fermée si la durée d'un tour est un multiple de la période, soit $\Omega = n{\omega _0}$.
4a/ Si $\rho = {\rho _m}[1 + \varepsilon (t)]$ alors l'équation en ε est :${\rho _m}\ddot \varepsilon - \left( {\frac{{\rho _0^4\omega _0^2}}{{\rho _m^3}}} \right)[1 - 3\varepsilon ] = - {\Omega ^2}({\rho _m} - R + {\rho _m}\varepsilon )$
4b/ La valeur moyenne correspond à ε = 0 : $ - \left( {\frac{{\rho _0^4\omega _0^2}}{{\rho _m^3}}} \right) = - {\Omega ^2}({\rho _m} - R)$on a
4c/ Par différence : ${\rho _m}\ddot \varepsilon + 3\left( {\frac{{\rho _0^4\omega _0^2}}{{\rho _m^3}}} \right)\varepsilon + {\Omega ^2}{\rho _m}\varepsilon = 0$ soit : $\ddot \varepsilon + 3\left( {\frac{{\rho _0^4\omega _0^2}}{{\rho _m^4}}} \right)\varepsilon + {\Omega ^2}\varepsilon = 0$
ce qui s'intègre en $\varepsilon (t) = {\varepsilon _0}\cos (\Omega 't + {\varphi _0})$ en posant : $\Omega ' = \sqrt {3\left( {\frac{{\rho _0^4\omega _0^2}}{{\rho _m^4}}} \right) + {\Omega ^2}} $.
Ce qui donne alors la vitesse angulaire: $\dot \theta = \frac{{\rho _0^2{\omega _0}}}{{{\rho ^2}}} \approx \frac{{\rho _0^2{\omega _0}}}{{\rho _m^2}}[1 - 2\varepsilon ]$.
4d/ Les trajectoires sont alors ses oscillations autour des sinusoïdes tracées sur un cylindre. La vitesse angulaire étant elle même oscillante.
5/ La pesanteur entaîne un mouvement de chute selon l'équation z = ½ gt2 qui s'ajoute aux oscillations. Au bout d'une période la "chute" vaut donc : $h = 2g{\pi ^2}/{\Omega ^2}$ .
On calcule alors : h = 5,6 mm, ce qui n'est pas négligeable.
___________________________

Concours commun Mines-Ponts (M, P', TA) 1990 Physique I (Corrigé)

Mines–Ponts, M, P’, TA, 1990 (Physique I)
Solutions proposées
    1. L’énergie interne \(U\) et l’entropie \(S\) vérifient respectivement \({\mathrm{d}}U = \delta W + \delta Q\) donc \({\mathrm{d}}U = C_L {\mathrm{d}}T + (F + h) {\mathrm{d}}L\) et \({\mathrm{d}}S = \frac{\delta Q}{T}\) donc \({\mathrm{d}}S = \frac{C_L}{T} {\mathrm{d}}T + \frac{h}{T} {\mathrm{d}}L\) pour des transformations réversibles; les expressions obtenues relient des différentielles de fonctions et variables d’état, donc ces expressions en sont pas limitées aux seules transformations réversibles et on peut écrire \({\frac{\partial U}{\partial T}} = C_L\), \({\frac{\partial U}{\partial L}} = F + h\), \({\frac{\partial S}{\partial T}} = \frac{C_L}{T}\) et \({\frac{\partial S}{\partial L}} = \frac{h}{T}\). On peut alors affirmer le lemme de Schwartz pour les deux fonctions \(U(T,L)\) et \(S(T,L)\), ce qui conduit aux deux relations \({\frac{\partial C_L}{\partial L}} = {\frac{\partial F}{\partial T}} + {\frac{\partial h}{\partial T}}\) et \({\frac{\partial }{\partial L}} \frac{C_L}{T} = {\frac{\partial }{\partial T}} \frac{h}{T}\), cette dernière pouvant être recopiée sous la forme \({\frac{\partial C_L}{\partial L}} = {\frac{\partial h}{\partial T}} - \frac{h}{T}\). L’identification des deux expressions mène à \(\frac{h}{T} = - {\frac{\partial F}{\partial T}}\), cette dernière dérivée partielle (calculée à \(L\) constant) s’identifie ici à \(\sigma\), d’où enfin le résultat \(h = - T \sigma\).

    2. Revenant aux expressions établies ci-dessus, \({\frac{\partial C_L}{\partial L}} = {\frac{\partial h}{\partial T}} - \frac{h}{T}\) s’écrit, puisque \(\sigma\) est constant, sous la forme \({\frac{\partial C_L}{\partial L}} = 0\).
    3. On revient enfin à l’expression \({\mathrm{d}}S = \frac{C_L}{T} {\mathrm{d}}T + \frac{h}{T} {\mathrm{d}}L\) écrite \({\mathrm{d}}S = \frac{C_L}{T} {\mathrm{d}}T - \sigma {\mathrm{d}}L\) qui s’intègre donc immédiatement en \(S(T,L) = S_m + C_L \ln \frac{T}{T_m} - \sigma (L - L_m)\).
    4. Si \(L\) augmente à température constante, \(\sigma > 0\) donc \(\Delta S_T = - \sigma \Delta L < 0\). Ce comportement est évidemment différent de celui d’un gaz parfait pour lequel l’identité thermodynamique \({\mathrm{d}}U = T {\mathrm{d}}S - P {\mathrm{d}}V\) et la première loi de Joule \({\mathrm{d}}U = 0\) si \({\mathrm{d}}T = 0\) permettent d’écrire \({\mathrm{d}}S = \frac{P}{T} {\mathrm{d}}V\) donc, au vu de l’équation d’état, \({\mathrm{d}}S = n R \frac{{\mathrm{d}}V}{V}\) soit \(\Delta S_T = n R \ln \frac{V_{\rm final}}{V_{\rm initial}} > 0\). Il n’y a rien de surprenant ici puisqu’une augmentation de volume implique une augmentation de l’indétermination sur les positions des molécules de gaz (augmentation du désordre) alors que l’augmentation de longueur d’un fil de caoutchouc se traduit par un alignement des molécules de polymère (diminution du désordre).
    5. À partir de la relation déjà posée \({\mathrm{d}}U = C_L {\mathrm{d}}T + (F + h){\mathrm{d}}L\) on peut, au vu de l’expression \(h = - \sigma T\), écrire \({\mathrm{d}}U = C_L {\mathrm{d}}T + \left[F_m - \sigma T_m + \rho (L - L_m)\right] {\mathrm{d}}L\) donc \(U = U_m + C_L(T - T_m) + \left(F_m - \sigma T_m\right) (L - L_m) + \frac{\rho}{2} (L - L_m)^2\) et \(\mathcal F = U - T S\) s’en déduit, après regroupement des termes et en fonction de \(\mathcal F_m = \mathcal F(T_m,L_m)\), il vient \(\mathcal F = \mathcal F_m + \left[C_L - S_m\right] (T-T_m) - C_L T \ln \frac{T}{T_m}+ \left[F_m - \sigma (T - T_m)\right](L - L_m) + \frac{\rho}{2} (L - L_m)^2\). On remarque alors que si \(T = T_m\), les variations de \(\mathcal F\) sont données par la relation \(\mathcal F - \mathcal F_m = \frac{\rho}{2} (L - L_m)^2\); plus généralement, on sait que \({\mathrm{d}}\mathcal F = \delta W - S {\mathrm{d}}T\) donc, pour une transformation isotherme, \({\mathrm{d}}\mathcal F\) s’identifie au travail reçu par le fil, opposé du travail qu’il fournit \(\delta W' = - {\mathrm{d}}E_p\); on peut donc identifier \(W = \Delta \mathcal F = \Delta E_p\) pour cette transformation réversible. La relation \(E_p = \frac{\rho}{2} (L - L_m)^2\) identifie un ressort élastique, de longueur à vide \(L_m\) et de raideur \(k = \rho\).
    6. Un cycle de Carnot est constitué de deux isothermes et de deux adiabatiques réversibles. L’équation d’une isotherme est \(F - F_m = \rho (L - L_m)\); c’est donc une droite de pente \(\rho\). L’équation d’une adiabatique réversible (isentropique) est \(S - S_m = C_L \ln \frac{T}{T_m} - \sigma (L - L_m)\) dont on doit éliminer \(T\) avec l’équation d’état \(F - F_m = \sigma (T - T_m) + \rho (L - L_m)\); il vient \(F = F_m + \rho(L - L_m) + \sigma T_m \left[\alpha \exp \left[ \frac{\sigma}{C_L} (L - L_m)\right] - 1\right]\) où on a posé \(\alpha = \exp \left(\frac{S - S_m}{C_L}\right)\). La pente de la courbe correspondante est \({\frac{{\mathrm{d}}F}{{\mathrm{d}}L}} = \rho + \frac{\alpha \sigma^2 T_m}{C_L} \exp \left[ \frac{\sigma}{C_L} (L - L_m)\right]\) donc \({\frac{{\mathrm{d}}F}{{\mathrm{d}}L}} > \rho\), ce qui permet de tracer l’allure du diagramme de Clapeyron (cf. figure). Le sens de parcours se déduit du caractère moteur du cycle: \(W = \oint F {\mathrm{d}}L < 0\) donc le parcours doit être à \({\mathrm{d}}L < 0\) pour les valeurs élevées de \(F\) (sens trigonométrique).

  1. Moteur d’Archibald.
      1. \(\overrightarrow{OA} = \overrightarrow{OC} + \overrightarrow{CA} = a \vec e_x + R \vec e_r\) en utilisant une base polaire d’angle \(\theta\); on en déduit donc (c’est en fait le théorème d’Al-Kashi) \(OA^2 = a^2 + 2 a R \cos \theta + R^2\) donc en se limitant au terme du premier ordre en \(a/R\), \(OA^2 = R^2 \left(1 + 2 \frac{a}{R} \cos \theta\right)\) soit \(OA = R + a \cos \theta\).
      2. La longueur la plus élevée est \(OA = R + a\), au moment où le fil se trouve à l’horizontale (\(\theta = 0\)) à l’entrée dans le bain d’eau chaude; la partie suivante du cycle \(A'B'\) correspond à l’évolution isotherme avec diminution de longueur (c’est donc une droite de pente \(\rho\)) jusqu’à atteindre \(\theta = \pi\) et la sortie de ce fil en \(B'\) du bain d’eau chaude. À cet instant on observe une phase de diminution rapide (quasiment instantanée dans le modèle choisi) de la température à longueur constante, donc une diminution de la force donnée par \(\Delta F = \sigma (T_2 - T_1) < 0\). Le fil est alors passé en \(C'\) dans l’air et subit une seconde évolution isotherme avec diminution de \(\theta\) de \(\pi\) à \(0\) qui l’amène en \(D'\), prêt à rentrer à nouveau dans le bain d’eau chaude; pendant la dernière transformation iso-longueur \(D'A'\), le fil subit une augmentation de la force donnée par \(\Delta F = \sigma (T_1 - T_2) > 0\). Le cycle est parcouru dans le sens (trigonométrique) moteur selon le schéma ci-dessous.
      1. La relation \(\delta Q = C_L {\mathrm{d}}T - T \sigma {\mathrm{d}}L\) appliquée à une transformation isotherme réversible \(A'B'\) fournit \(Q_{A'B'} = 2 a T_1 \sigma\); il faut ajouter à ce terme de transfert thermique lors du chauffage iso-longueur \(D'A\) pour lequel on peut supposer une transformation réversible et écrire \(Q_{D'A'} = C_L (T_1 - T_2)\) et on obtient \(Q_1 = C_L (T_1 - T_2) + 2 a T_1 \sigma\).
        Remarquons que, sans supposer une transformation réversible entre \(D'\) et \(A'\), l’expression de \(U\) établie plus haut permet d’écrire \(\Delta U = C_L \Delta T\) à \(L\) fixé donc \(Q = \Delta U - W\) s’écrit encore \(Q = \Delta U\) si on se souvient que, même pour une évolution irréversible, \(W = \int F_{\rm ext}{\mathrm{d}}L\) est nul à longueur constante; finalement, le résultat affirmé ci-dessus ne dépend pas de l’hypothèse de réversibilité du changement de température iso-longueur \(D'A'\).
      2. Les mêmes raisonnements amènent immédiatement à \(Q_2 = C_L (T_2 - T_1) - 2 a T_2 \sigma\). Le premier principe de la thermodynamique s’écrit, pour un cycle, \(\Delta U = 0 = Q_1 + Q_2 - W\) si \(W\) est le travail fourni par le fil lors d’un tour; on a donc \(W = 2 a \sigma (T_1 - T_2)\).
      3. Le travail fourni est égal à l’aire du cycle; l’aire du parallélogramme tracé est le produit de la largeur \(2a\) par la hauteur \(\sigma (T_1 - T_2)\), ce qui confirme le résultat \(W = 2 a \sigma (T_1 - T_2)\).
      4. Lorsque le point \(A\) se trouve à la position définie par l’angle \(\theta\), le moment de la force \(\vec F\) est défini par \(\vec{\mathcal M} = \overrightarrow{CA} \wedge \vec F\)\(\vec F = F \vec u\) est dirigé de \(A\) vers \(O\); on écrit alors \(\overrightarrow{CA} = R \vec e_r\) tandis que \(\overrightarrow{AO} = - a \vec e_x - R \vec e_r\) donc le vecteur unitaire dirigé de \(A\) vers \(O\) s’écrit \(\vec u = - \frac{a \vec e_x + R \vec e_r}{R \left(1 + \frac{a}{R} \cos \theta\right)}\) donc, à l’ordre le plus bas, \(\vec{\mathcal M} = a F \vec e_z\) qu’on écrit \(\vec{\mathcal M} =\mathcal M \vec e_z\) avec \(\mathcal M = a F(T,L)\).
        Le moment exercé sur la roue lors du demi-tour entre \(\theta = 0\) et \(\theta = \pi\) (le fil étant dans l’eau chaude) est donc \(\mathcal M_1 = a \left[F_m + \sigma (T_1 - T_m) + \rho (R + a \cos \theta - L_m)\right]\) et le travail reçu par la roue pendant ce demi-tour est \(W_1 = \int_0^\pi \mathcal M_1 {\mathrm{d}}\theta\). De la même manière, lors du demi-tour ultérieur (le fil étant alors dans l’air), \(\mathcal M_2 = a \left[F_m + \sigma (T_2 - T_m) + \rho (R + a \cos \theta - L_m)\right]\) et \(W_2 = \int_{\pi}^0 \mathcal M_2 {\mathrm{d}}\theta\). L’intégration sur un tour complet conduit à \(W = \int_0^{\pi} \left(\mathcal M_1 - \mathcal M_2\right) {\mathrm{d}}\theta\) donc \(W = \int_0^{\pi} a \sigma (T_1 - T_2) \cos \theta {\mathrm{d}}\theta\) soit ici encore \(W = 2 a \sigma (T_1 - T_2)\).
      5. La définition générale du rendement d’un moteur thermique conduit à poser \(\eta = \frac{W}{Q_1}\) donc on en déduit immédiatement \(\eta = \frac{2 a \sigma (T_1 - T_2)}{C_L (T_1 - T_2) + 2 a T_1 \sigma}\).
      1. Le rendement du cycle moteur de Carnot s’obtient en écrivant les premier et second principes sous la forme \(0 = Q_1 + Q_2 - W\) (comme pour le cycle de Stirling) et \(\frac{Q_1}{T_1} + \frac{Q_2}{T_2} = 0\) (l’inégalité de Clausius est une égalité dans le cas d’une évolution complètement réversible) donc \(\eta_C = 1 + \frac{Q_2}{Q_1}\) s’écrit aussi \(\eta_C = 1 - \frac{T_2}{T_1}\) ce qui mène à \(\alpha = \frac{1}{1 + \displaystyle \frac{C_L}{2a\sigma}(1 - \displaystyle \frac{T_2}{T_1})}\) qu’on peut aussi écrire \(\alpha = \frac{1}{1 + \displaystyle \frac{C_L \eta_C}{2a\sigma}}\). On remarque bien sûr que \(\alpha < 1\) donc \(\eta < \eta_C\), conformément au théorème de Carnot (voir aussi la question suivante).
      2. On a vu que \(Q_{D'A'} = C_L (T_1 - T_2)\), et de même \(Q_{B'C'} = C_L (T_2 - T_1)\) donc \(Q_{B' \to C' \cup D' \to A'} = 0\) donc cet ensemble est adiabatique. Toutefois, un cycle de Carnot doit être complètement adiabatique; les deux évolutions de température en \(B'C'\) et \(D'A'\) sont irréversibles donc le rendement dans un cycle de Stirling est plus faible que le rendement dans un cycle de Carnot.

      1. L’expression ci-dessus du rendement mène à \(\eta = {1,2 \cdot 10^{-5}}\) (valeur intensive donc indépendante du nombre de fils). La puissance moyenne du moteur est \(\mathcal P = 2N \frac{W}{\tau}\) où la durée d’un cycle est \(\tau = \frac{2\pi}{\omega}\) (valeur extensive donc proportionnelle au nombre de fils fonctionnant en même temps); numériquement, \(\mathcal P = \frac{2N\omega a \sigma (T_1 - T_2)}{\pi} = 0,51{\,\mathrm{W}}\).
      2. Le débit \(\mathcal D_m\) s’identifie à la masse d’eau remontée par unité de temps d’une hauteur \(h\), avec donc une puissance mécanique consommée \(\mathcal P > \mathcal D_m g h\); on a donc \(\mathcal D_m < \frac{\mathcal P}{g h} = {5,2 \cdot 10^{-3}}{\,\mathrm{kg}\cdot\mathrm{s}^{-1}}\) en prenant \(g = 9,8{\,\mathrm{m}\cdot\mathrm{s}^{-2}}\); c’est une valeur assez élevée (\(310\) litres par minute) ce qui conduit à recommander l’utilisation de ce appareil, la valeur très faible du rendement étant compensée par le caractère gratuit de la source thermique (on peut imaginer le chauffage de l’eau par le rayonnement solaire par exemple).

Autres Concours

2011  : Concours ENAC de  physique 2011  :  énoncé ,  corrigé Concours ICNA de  physique 2011  :  énoncé ,  corrigé Concours ICNA de ...